Re: [obm-l] problema de probabilidade

2022-11-09 Por tôpico Claudio Buffara
Essa também: https://thedailyviz.com/2016/09/17/how-common-is-your-birthday-dailyviz/ On Wed, Nov 9, 2022 at 12:04 PM Claudio Buffara wrote: > Achei isso aqui interessante: https://www.panix.com/~murphy/bday.html > > []s, > Claudio. > > On Tue, Nov 8, 2022 at 9:56 PM Ralph Costa Teixeira >

Re: [obm-l] problema de probabilidade

2022-11-09 Por tôpico Claudio Buffara
Achei isso aqui interessante: https://www.panix.com/~murphy/bday.html []s, Claudio. On Tue, Nov 8, 2022 at 9:56 PM Ralph Costa Teixeira wrote: > Mis ou menos... O que faltou foi a hipótese exata da distribuição de > probabilidade dos aniversários. > > Se a gente supõe que cada mês tem os

Re: [obm-l] problema de probabilidade

2022-11-09 Por tôpico Anderson Torres
Em ter, 8 de nov de 2022 21:55, Ralph Costa Teixeira escreveu: > Mis ou menos... O que faltou foi a hipótese exata da distribuição de > probabilidade dos aniversários. > > Se a gente supõe que cada mês tem os mesmos 1/12 de chance para cada > aluno, e que os meses são independentes entre si,

Re: [obm-l] problema de probabilidade

2022-11-08 Por tôpico Ralph Costa Teixeira
Mis ou menos... O que faltou foi a hipótese exata da distribuição de probabilidade dos aniversários. Se a gente supõe que cada mês tem os mesmos 1/12 de chance para cada aluno, e que os meses são independentes entre si, sim, p=12/12^2=1/12~8.3%. Agora, talvez um modelo um pouco mais

Re: [obm-l] Problema da IMO

2020-09-22 Por tôpico Anderson Torres
Em sáb., 12 de set. de 2020 às 01:18, Pedro José escreveu: > > Boa noite! > Atrapalhou meu vinho e o filme que estava assistindo mas consegui. Não gostei > tanto, agora que consegui, é muito trabalhoso. > > 2= [3(y+1)(z+1)-1]/2yz > yz= 3(yz+2) (i) > z(y-3)= 3y +2 (ii) > y(z-3)=3z+2 (iii) >

Re: [obm-l] Problema da IMO

2020-09-11 Por tôpico Pedro José
Boa noite! Atrapalhou meu vinho e o filme que estava assistindo mas consegui. Não gostei tanto, agora que consegui, é muito trabalhoso. 2= [3(y+1)(z+1)-1]/2yz yz= 3(yz+2) (i) z(y-3)= 3y +2 (ii) y(z-3)=3z+2 (iii) (i)*(ii) yz(z-3)(y-3)= 9yz+6(y+z)+4 e Voilá: (z-3)(y-3)=11. Saudações, PJMS Em

Re: [obm-l] Problema da IMO

2020-09-11 Por tôpico Pedro José
Boa noite! Fui em uma linha parecida com a primeira solução, embora não visse necessidade de mudança de variáveis. Mas o b achei sempre por restrição. Esse "it implies" e aparece um número fatorado, não consegui captar, embora tenha gostado do recurso, já que é bem restritivo. Sudações, PJMS Em

Re: [obm-l] Problema da IMO

2020-09-11 Por tôpico Pedro José
Boa noite! Grato, Ralph! Estou estudando a solução. Pelo menos, não me decepcionei. A resposta estava correta, Saudações. PJMS Em sex., 11 de set. de 2020 às 22:33, Ralph Costa Teixeira < ralp...@gmail.com> escreveu: > Essa eh da IMO 1992. Tem uma solucao aqui: >

Re: [obm-l] Problema da IMO

2020-09-11 Por tôpico Ralph Costa Teixeira
Essa eh da IMO 1992. Tem uma solucao aqui: http://sms.math.nus.edu.sg/Simo/IMO_Problems/92.pdf On Fri, Sep 11, 2020 at 10:06 PM Pedro José wrote: > Bom dia! > > Recebi de um filho de um amigo, um problema que já o fizera. > (a-1)(b-1)(c-1) | abc-1; 1 > Confesso que desta feita gastei mais

Re: [obm-l] Problema simples gera um complicado?

2020-07-02 Por tôpico Professor Vanderlei Nemitz
Muito obrigado, Matheus! Vou estudar sobre esse ponto especial! Em qui., 2 de jul. de 2020 às 19:58, Matheus Bezerra < matheusbezerr...@gmail.com> escreveu: > Olá Vanderlei, boa noite. Esse é um fato conhecido, essas retas concorrem > em um ponto chamado de Ponto de Fermat. Pesquisa por isso que

Re: [obm-l] Problema simples gera um complicado?

2020-07-02 Por tôpico Matheus Bezerra
Olá Vanderlei, boa noite. Esse é um fato conhecido, essas retas concorrem em um ponto chamado de Ponto de Fermat. Pesquisa por isso que você deve encontrar alguma prova. ;) *Matheus BL* Em qui., 2 de jul. de 2020 às 18:55, Professor Vanderlei Nemitz < vanderma...@gmail.com> escreveu: > Oi,

Re: [obm-l] Problema de Geometria plana

2020-05-11 Por tôpico Projeto Iteano
De fato, se vc desenhar com régua e compasso dá pra ver q n é verdade Em seg, 11 de mai de 2020 20:35, Vanderlei Nemitz escreveu: > Boa noite! > Vi esse problema em uma lista, mas talvez tenha alguma falha no enunciado. > Ou será no leitor? > Muito obrigado! > > *Seja ABC um triângulo e D um

Re: [obm-l] Problema

2020-03-17 Por tôpico Pedro José
Boa noite! Você já formulou esse problema em set/2019 e Daniel Jelin apresentou uma bela solução. Saudações, PJMS Em ter, 17 de mar de 2020 19:26, escreveu: > Problema > Um mágico e seu assistente realizam um truque da maneira seguinte. Existem > 12 caixas vazias e fechadas, colocadas em fila.

Re: [obm-l] Problema 5 OBMU 2018

2019-10-31 Por tôpico Claudio Buffara
Mudando um pouco a notação... Ponha: Df(x) = f(x+1) - f(x). Para todo x em R+, e todo inteiro positivo k, existe (pelo TVM) y_k entre x e x+1 tal que (Df)^(k)(x) = f^(k)(x+1) - f^(k)(x) = f'^(k+1)(y_k) > 0. Logo, Df satisfaz a primeira condição do enunciado. Além disso, como f' é positiva para

Re: [obm-l] Problema 5 OBMU 2018

2019-10-30 Por tôpico Ernesto Rodrigues
Pense um pouco sobre g(x)=f(x+1)-f(x), essa questão é bem tricky, o segredo é que a g satisfaz as condições da questão, logo, por indução, vale que g(n) é maior ou igual a dois elevado a n menos um, mas isto implica que o mesmo vale para f(n+1), completando a indução (tem que pensar bastante para

Re: [obm-l] PROBLEMA

2019-09-04 Por tôpico Daniel Jelin
queremos fazer com que cada umas das 12 caixas indique um conjunto único de outras 4 caixas (aquelas que o mágico irá abrir) de tal modo que o par de caixas que contenham as moedas seja uma das 6 combinações dos 4 elementos, 2 a 2, desse conjunto. vamos imaginar as caixas numeradas de 1 a 12. são

Re: [obm-l] Problema sobre Derivadas

2019-08-30 Por tôpico Luiz Antonio Rodrigues
Olá, Claudio! Olá, Gabriel! Muito obrigado pela ajuda! Tudo ficou claro agora! Abraços Luiz On Fri, Aug 30, 2019, 3:15 PM Claudio Buffara wrote: > h'(x) = g'(f(x))*f'(x) ==> h'(3) = g'(f(3))*f'(3) = g'(5)*3 = 4*3 = 12. > > Imagino que a sua dificuldade esteja em como aplicar a regra da cadeia,

Re: [obm-l] Problema sobre Derivadas

2019-08-30 Por tôpico Claudio Buffara
h'(x) = g'(f(x))*f'(x) ==> h'(3) = g'(f(3))*f'(3) = g'(5)*3 = 4*3 = 12. Imagino que a sua dificuldade esteja em como aplicar a regra da cadeia, que nos livros de cálculo é normalmente enunciada como: dy/dx = dy/du * du/dx (*) sem especificar quem são os argumentos (variáveis independentes)

Re: [obm-l] Problema sobre Derivadas

2019-08-30 Por tôpico Gabriel Lopes
Ola, boa tarde. Isso é uma simples aplicação da regra da cadeia. H'(x) = g'(f (x))*f'(x) H'(3) = g'(f (3))*f'(3) = g (5) * 3 = 9 Em Sex, 30 de ago de 2019 14:16, Luiz Antonio Rodrigues < rodrigue...@gmail.com> escreveu: > Olá, pessoal! > Boa tarde! > Tudo bem? > Estou confuso com o problema

[obm-l] Re: [obm-l] Problema da Olimpíada Brasileira de Matemática para Universitários

2019-07-12 Por tôpico Pedro Angelo
Pensando rápido aqui. Dados discos D1 e D2, queremos pontos P1 e P2 tais que toda parábola que passa por P1 e P2 toca pelo menos um dos discos. (Estou assumindo que P1 e P2 estão proibidos de pertencerem aos discos, pois caso contrário bastaria escolher Pj em Dj.) Obviamente, P1 e P2 devem estar

[obm-l] Re: [obm-l] Problema de Matemática Financeira

2019-04-01 Por tôpico Luiz Antonio Rodrigues
Olá, Pedro! Seguirei seu conselho: vou conversar com alguém que entenda bastante do assunto. Muito obrigado e um abraço! Luiz On Sun, Mar 31, 2019, 7:23 PM Pedro José wrote: > Boa noite! > > Mas tem de verificar se é praxe fazer assim ou não. Nos juros compostos, > você pode trabalhar com

[obm-l] Re: [obm-l] Re: [obm-l] Re: [obm-l] Problema de Matemática Financeira

2019-03-31 Por tôpico Pedro José
Boa noite! Mas tem de verificar se é praxe fazer assim ou não. Nos juros compostos, você pode trabalhar com qualquer referência no tempo e depois levar para uma mesma que dá a mesma coisa. Juro simples não. Ma ninguém trabalha com juro simples. Tem que ver uma pessoa que entenda de financeira.

[obm-l] Re: [obm-l] Re: [obm-l] Problema de Matemática Financeira

2019-03-31 Por tôpico Luiz Antonio Rodrigues
Olá, Pedro! Tudo bem? Concordo com suas observações. Eu havia chegado no valor calculado no item (1). Mas eu entendi os cálculos dos itens (2) e (3). Agora sim eu percebi qual deve ser o raciocínio para resolver o problema! Muito obrigado pela ajuda! Um abraço! Luiz On Sun, Mar 31, 2019, 1:04 PM

[obm-l] Re: [obm-l] Problema de Matemática Financeira

2019-03-31 Por tôpico Pedro José
Bom dia! Primeiramente, nenhuma instituição empresta a juros simples. Segundo, nenhuma instituição permite que o pagamento fique a vontade do cliente. Há mora para esse caso. Não consigo entender a natureza desses problemas. Não entendo muito de matemática financeira. Mas o cálculo à taxa de

[obm-l] Re: [obm-l] Re: [obm-l] [Problema da Balança]

2019-03-26 Por tôpico gilberto azevedo
Tentei muito assim, não saiu. Gabarito consta n - 1 mesmo. Em ter, 26 de mar de 2019 22:47, Gabriel Lopes escreveu: > Para mim o numero de pesagem mínimal é n-1, para n maior ou igual a 3, > para se obter tanto o maximo quanto o minimo,( faça indução) .Para obter o > maximo e depois o mínimo

[obm-l] Re: [obm-l] [Problema da Balança]

2019-03-26 Por tôpico Gabriel Lopes
Para mim o numero de pesagem mínimal é n-1, para n maior ou igual a 3, para se obter tanto o maximo quanto o minimo,( faça indução) .Para obter o maximo e depois o mínimo separe o o menor na primeira pesagem e prossiga para obter o maximo n-1 mais n-2 pesagens, acho q é isso Em Ter, 26 de mar

[obm-l] Re: [obm-l] Re: [obm-l] Re: [obm-l] Problema olimpíada de maio

2019-01-23 Por tôpico Ralph Teixeira
Sim, nao vi porque que algum resto apareceria mais do que os outros... Achei que eu conseguiria uma funcao que levasse cada classe de restos numa outra, mas soh consegui pareamentos. Com os dois paremntos, deu. On Wed, Jan 23, 2019 at 10:27 AM Mauricio de Araujo < mauricio.de.ara...@gmail.com>

[obm-l] Re: [obm-l] Re: [obm-l] Problema olimpíada de maio

2019-01-23 Por tôpico Mauricio de Araujo
Bela solução!! mas qual foi o teu insight? Desconfiança de que havia uma distribuição uniforme dos restos possíveis? Att. Em qua, 23 de jan de 2019 às 00:47, Ralph Teixeira escreveu: > Hm, tive uma ideia, confiram se funciona. > > Seja S o conjunto dos numeros obtidos pela permutacao dos

[obm-l] Re: [obm-l] Problema olimpíada de maio

2019-01-22 Por tôpico Ralph Teixeira
Hm, tive uma ideia, confiram se funciona. Seja S o conjunto dos numeros obtidos pela permutacao dos digitos de 1 a 7, e seja x_i a quantidade de elementos de S que deixam resto i na divisao por 7 (i=0,1,2,3,4,5,6). Agora vamos fazer dois pareamentos. (Ou seja, vamos criar funcoes f,g:S->S tal

Re: [obm-l] Problema de Geometria

2019-01-01 Por tôpico Pacini Bores
Como disse anteriormente, o enunciado está com problemas. Pacini Em 31/12/2018 23:19, Pacini Bores escreveu: > Oi Marcelo, > > Está me parecendo que fixando o vértice B e variando o vertice C nas > condições do problema , que o ângulo pedido está variando Pode ser que > eu esteja

Re: [obm-l] Problema de Geometria

2019-01-01 Por tôpico Pedro José
Boa noite! Na verdade, se B>76 não tem resposta. O ponto E ficaria externo ao lado BC. Teria que mudar o problema para E pertencente a l(B,C). Mas assim mesmo o ânfulo CDE não seria constante. Saudações, PJMS Em ter, 1 de jan de 2019 14:13, Pedro José Boa tarde! > Você tem certeza que o

Re: [obm-l] Problema de Geometria

2019-01-01 Por tôpico Pedro José
Boa tarde! Você tem certeza que o problema é esse. Se C=84 e B=48, dá 42. Se C=100 e B= 32, dá 66. Se B >= 90 não tem resposta. Saudações, PJMS Em seg, 31 de dez de 2018 20:12, Marcelo de Moura Costa Caros colegas, me deparei com um problema que até então não estou > enxergando uma solução,

Re: [obm-l] Problema de Geometria

2018-12-31 Por tôpico Pacini Bores
Oi Marcelo, Está me parecendo que fixando o vértice B e variando o vertice C nas condições do problema , que o ângulo pedido está variando Pode ser que eu esteja errado, vou verificar!!! Pacini Em 31/12/2018 20:03, Marcelo de Moura Costa escreveu: > Caros colegas, me deparei com um

Re: [obm-l] Problema

2018-11-26 Por tôpico benedito
2018 17:52:58 Assunto: Re: [obm-l] Problema Tentei um tabuleiro 12x12 e consegui uma configuração que não tem nenhuma lâmpada ruim. Acho que dá para estender o padrão para um 2017x2017. Mas me parece que a paridade importa e talvez o caso 2017x2017 tenha um mínimo de uma lâmpada ru

Re: [obm-l] Problema

2018-11-26 Por tôpico Bruno Visnadi
Tentei um tabuleiro 12x12 e consegui uma configuração que não tem nenhuma lâmpada ruim. Acho que dá para estender o padrão para um 2017x2017. Mas me parece que a paridade importa e talvez o caso 2017x2017 tenha um mínimo de uma lâmpada ruim. https://i.imgur.com/HhWrZzu.png Em seg, 26 de nov de

Re: [obm-l] Problema

2018-11-26 Por tôpico Claudio Buffara
Sem pensar muito no problema, aqui vai uma sugestão: tente com um tabuleiro menor, 4x4 ou 5x5, pra ver se acha algum padrão. []s, Claudio. On Mon, Nov 26, 2018 at 9:52 AM wrote: > Alguém pode me dar uma sugestão para o problema seguinte? > > *Problema* > Há uma lâmpada em cada casa de um

Re: [obm-l] Problema Simples de Probabilidade

2018-11-23 Por tôpico Luiz Antonio Rodrigues
Olá, Pedro! Tudo bem? Muito obrigado pela resposta! Um abraço! Luiz On Thu, Nov 22, 2018, 7:13 PM Pedro José Boa noite! > > Considerando o modelo equiprovável, já que não há menção ao contrário. > Resolvi de outra maneira e também deu 13/35. > > Caminhos possíveis: PBB, BPB e BBQ ==>

Re: [obm-l] Problema Simples de Probabilidade

2018-11-22 Por tôpico Pedro José
Boa noite! Considerando o modelo equiprovável, já que não há menção ao contrário. Resolvi de outra maneira e também deu 13/35. Caminhos possíveis: PBB, BPB e BBQ ==> 2*(4*3*2)/(7*6*5) + (3*2)/(7*6)= 13/35 P preta, B branca Q qualquer Menor do que 1/2, o que é esperado, uma vez que há mais bolas

Re: [obm-l] Problema Simples de Probabilidade

2018-11-22 Por tôpico Luiz Antonio Rodrigues
Olá, Ralph! Bom dia! Cheguei neste resultado também! Conclusão: gabarito incorreto! Muito obrigado pela ajuda! Um abraço! Luiz On Thu, Nov 22, 2018, 1:58 AM Ralph Teixeira Bolas B1,B2,B3,P1,P2,P3,P4. > > Ha C(7,3)=35 maneiras igualmente provaveis de retirar 3 bolas > simultaneamente (ignoro a

Re: [obm-l] Problema Simples de Probabilidade

2018-11-21 Por tôpico Ralph Teixeira
Bolas B1,B2,B3,P1,P2,P3,P4. Ha C(7,3)=35 maneiras igualmente provaveis de retirar 3 bolas simultaneamente (ignoro a ordem). Destas, tem C(3,2).C(4,1)+C(3,3).C(4,0) = 12+1=13 maneiras de tirar pelo menos 2 brancas (12 maneiras de tirar 2 brancas e 1 reta, mais uma de tirar 3 brancas). Entao eu

Re: [obm-l] Problema 6 - IMO 2001

2018-11-15 Por tôpico Pedro José
Boa tarde! Equivoquei-me quando deduzi a fórmula da diagonal do quadrilátero. Considerei x o ângulo BAD e y o ângulo ABC mas coloquei senx/seny = AC/BD, quando era o inverso. Na verdade onde AC é AB e vice-versa. Até porque BD é que permanece constante em qualquer ordem e não AC. BD^2=a^2-ac+c^2.

Re: [obm-l] Problema 6 - IMO 2001

2018-11-15 Por tôpico Pedro José
Boa tarde! Em tempo, a ordem usada dos vértices foi A, B, C, D, no sentido trigonométrico. Só variou a nomemclatura da medida dos lados. Saudações, PJMS Em Qui, 15 de nov de 2018 13:03, Pedro José escreveu: > Boa tarde! > Não tinha atinado que no segundo caso, o fator (ab+cd) está no numerador

Re: [obm-l] Problema 6 - IMO 2001

2018-11-15 Por tôpico Pedro José
Boa tarde! Não tinha atinado que no segundo caso, o fator (ab+cd) está no numerador do valor do quadrado de ambas diagonais. Realmente serve de qualquer jeito. (i) a, b, d, c no sentido trigonométrico. (ad+bc)*(ab+cd) =AC^2*(ac+bd) (ii) a, d, b, c no mesmo sentido. (ab+cd)*(ac+bd)=BD^2*(ad+bc)

Re: [obm-l] Problema 6 - IMO 2001

2018-11-15 Por tôpico Anderson Torres
Em qua, 14 de nov de 2018 16:53, Pedro José Boa tarde! > > Porém, me ficou uma dúvida! Como definir a ordem dos lados, os de medidas > a e c devem ser adjacentes, assim como os de medida b e d. > Mas como definir se os de a e b ou de a e d são adjacentes??? > Bem, tecnicamente qualquer um

Re: [obm-l] Problema 6 - IMO 2001

2018-11-14 Por tôpico Jeferson Almir
A ordem segue a,d,b,c no sentido horário devido a relação a^2 -ac + c^2 = b^2 + bd + d^2 Em qua, 14 de nov de 2018 às 15:53, Pedro José escreveu: > Boa tarde! > > Porém, me ficou uma dúvida! Como definir a ordem dos lados, os de medidas > a e c devem ser adjacentes, assim como os de medida b e

Re: [obm-l] Problema 6 - IMO 2001

2018-11-14 Por tôpico Pedro José
Boa tarde! Porém, me ficou uma dúvida! Como definir a ordem dos lados, os de medidas a e c devem ser adjacentes, assim como os de medida b e d. Mas como definir se os de a e b ou de a e d são adjacentes??? Grato, PJMS Em ter, 13 de nov de 2018 às 13:44, Pedro José escreveu: > Bom dia! > >

Re: [obm-l] Problema 6 - IMO 2001

2018-11-13 Por tôpico Pedro José
Bom dia! Depois da observação do Anderson Torres é que atinei o quanto é bonita a sua solução se você prosseguir. Sua preocupação não deve ser em relação ao produto AC*BD, nem com os valores AC ou BD; mas sim que tanto BD^2, como AC^2 são inteiros. Falta uma beirinha e a solução indicada pelo

Re: [obm-l] Problema 6 - IMO 2001

2018-11-12 Por tôpico Anderson Torres
Você quase resolveu! Posso dizer que esta era basicamente a solução oficial. Tente mais um pouco, que o caminho é esse. Em 8 de nov de 2018 23:27, "Jeferson Almir" escreveu: Pessoal peço ajuda no problema : Sejam a, b , c , d inteiros e a > b > c > d > 0 . Suponha que ac + bd = ( b+ d + a - c

[obm-l] Re: [obm-l] Re: [obm-l] Re: [obm-l] [Problema] Achar o mínimo do valor absoluto de uma soma complexa

2018-11-12 Por tôpico Bernardo Freitas Paulo da Costa
Oi, acho que você interpretou o enunciado de forma a "evitar os complexos". O problema original fala de "achar um ponto dentro do círculo", então talvez não sejam apenas os pontos na circunferência (como parece que a sua solução faz, ao ordenar todos pelos ângulos centrais), mas qualquer ponto

Re: [obm-l] Problema 6 - IMO 2001

2018-11-09 Por tôpico Claudio Buffara
Ou olhe aqui: https://mks.mff.cuni.cz/kalva/imo/isoln/isoln016.html On Fri, Nov 9, 2018 at 12:11 AM Bruno Visnadi wrote: > Não entendi. Se a, b, c e d são inteiros, ac e bd certamente são racionais. > > Em qui, 8 de nov de 2018 às 22:27, Jeferson Almir < > jefersonram...@gmail.com> escreveu: >

Re: [obm-l] Problema 6 - IMO 2001

2018-11-08 Por tôpico Bruno Visnadi
Não entendi. Se a, b, c e d são inteiros, ac e bd certamente são racionais. Em qui, 8 de nov de 2018 às 22:27, Jeferson Almir escreveu: > Pessoal peço ajuda no problema : > > Sejam a, b , c , d inteiros e a > b > c > d > 0 . > Suponha que > ac + bd = ( b+ d + a - c )( b+ d -a + c ) > > Mostre

Re: [obm-l] Problema de Trigonometria

2018-11-07 Por tôpico Luiz Antonio Rodrigues
Olá, Ralph! Bom dia! Muito obrigado pela ajuda! Agora o problema faz sentido! Um abraço! Luiz On Tue, Nov 6, 2018, 10:45 PM Ralph Teixeira Eles disseram que a expressão eh uma identidade **em x**. Abrindo a > expressão da direita e organizando, o que foi dado eh que: >

Re: [obm-l] Problema de Trigonometria

2018-11-06 Por tôpico Ralph Teixeira
Eles disseram que a expressão eh uma identidade **em x**. Abrindo a expressão da direita e organizando, o que foi dado eh que: sinx+2cosx=(Asiny)sinx+(Acosy)cosx vale para todo x real. Como A e y sao NUMEROS (nao dependem de x), o unico jeito de isso acontecer eh se os coeficientes de sinx e cosx

[obm-l] Re: [obm-l] Re: [obm-l] [Problema] Achar o mínimo do valor absoluto de uma soma complexa

2018-11-05 Por tôpico Pedro José
Boa tarde! Engano P4 e não Pe é o que engloba mais pontos. E temos que somar 1 a ca engloba, pois esqueci de contar o próprio ponto. Mas não influencia para o que englobe o máximo. Saudações, PJMS Em seg, 5 de nov de 2018 às 16:41, Pedro José escreveu: > Boa tarde! > Se entendi o que você

[obm-l] Re: [obm-l] Re: [obm-l] [Problema] Achar o mínimo do valor absoluto de uma soma complexa

2018-11-05 Por tôpico Pedro José
Boa tarde! Se entendi o que você quer, não entendi qual a relação com o mínimo de uma soma complexa? Para resolver o problema que você propõe, entendi: (i) a excursão como a geração de um setor circular, a partir de um ponto inicial, essa incursão tem dois sentidos, trigonométrico ou

[obm-l] Re: [obm-l] [Problema] Achar o mínimo do valor absoluto de uma soma complexa

2018-11-03 Por tôpico Bruno Visnadi
Não entendi a pergunta - o que é uma excursão? Em sáb, 3 de nov de 2018 às 22:18, Jardiel Cunha escreveu: > Olá! > > > Estou trabalhando em um projeto e um problema está me tirando o sono há > algum tempo. Meu trabalho é na área de engenharia de microondas. A solução > que eu encontrei até

Re: [obm-l] problema de contagem

2018-11-03 Por tôpico Anderson Torres
Em ter, 30 de out de 2018 às 15:41, Luís Lopes escreveu: > > Sauda,c~oes, > > > Construir um triângulo dados três quaisquer dos seguintes elementos: > > > A,B,C - ângulos > > a,b,c - lados > > h_a,h_b,h_c - alturas > > m_a,m_b,m_c - medianas > > d_a,d_b,d_c - bissetrizes internas > > e_a,e_b,e_c

[obm-l] Re: [obm-l] Problema Olímpico - Nível 1 - Segunda fase - 2011

2018-10-16 Por tôpico Pedro José
Boa tarde! Desculpe-me, acabei não prestando atenção no seu questionamento:"*Inclusive, como está o desenho, são 47 pessoas respondendo "sim", e não 48 como hipótese inicial. Concordam**?*" Discordo pois não é uma fila é um círculo e o V76, estará a direita de A1, então teremos de A1 a A24

[obm-l] Re: [obm-l] Problema Olímpico - Nível 1 - Segunda fase - 2011

2018-10-16 Por tôpico Pedro José
Boa tarde! Não vejo erro na solução do sítio da OBM. 1) Entendi sua referência a início, como o primeiro entrevistado. 2) Realmente, não há nenhuma diferenciação entre se começar com azul ou por vermelho. Não há restrição para que as respostas "SIM" sejam consecutivas. Portanto, se você pegar

Re: [obm-l] Problema da OBM 2017

2018-05-10 Por tôpico Kevin Felipe Kuhl Oliveira
Boa tarde, Vanderlei Bom, o que pensei nessa letra é o seguinte: Temos que encontrar o elemento que ocupa a posição 2017 (no conjunto crescentemente ordenado dos números que podemos escrever na terra dos Impas). Para isso, podemos pensar qual o número mínimo de algarismos que esse número

Re: [obm-l] Re: [obm-l] Re: [obm-l] Re: [obm-l] Problema de minimização

2018-03-12 Por tôpico Claudio Buffara
A propriedade de reflexão na elipse é outra consequência interessante da desigualdade triangular e, mais precisamente, da solução do problema de achar o caminho mais curto entre os pontos A e B tocando uma reta dada (A e B estando num mesmo semiplano determinado pela reta). No fim, o caminho

[obm-l] Re: [obm-l] Re: [obm-l] Re: [obm-l] Problema de minimização

2018-03-12 Por tôpico Anderson Torres
Em 11 de março de 2018 22:37, Ralph Teixeira escreveu: > ...e portanto a elipse de focos A e B passando por O tem que ser tangente aa > elipse de focos C e D passando por O Fica como exercicio pensar o que > uma coisa tem a ver com a outra. Heuristicamente, eu chutaria que

[obm-l] Re: [obm-l] Re: [obm-l] Problema de minimização

2018-03-11 Por tôpico Ralph Teixeira
...e portanto a elipse de focos A e B passando por O tem que ser tangente aa elipse de focos C e D passando por O Fica como exercicio pensar o que uma coisa tem a ver com a outra. (O que podia ser visto de outras formas, diga-se de passagem, se voce sabe que a normal a tal elipse eh a

Re: [obm-l] Re: [obm-l] Problema de minimização

2018-03-11 Por tôpico Claudio Buffara
É isso aí! Uma aplicação simples mas elegante da desigualdade triangular. E o ponto O não parece ser tão difícil de conjecturar. Afinal, o ponto de intersecção das diagonais talvez seja o “ponto notável” mais óbvio de um quadrilátero (certamente é o mais fácil de construir - duas aplicações da

[obm-l] Re: [obm-l] Problema de minimização

2018-03-11 Por tôpico Douglas Oliveira de Lima
Seja o quadrilátero ABCD cujas diagonais são AC e BD, e O o ponto de intersecção das diagonais. Seja também um ponto P em seu interior e as distâncias PA, PB, PC, PD, temos por desigualdade triângular que PA+PC>=AC e PB+PD>=BD. Claramente vemos que o ponto P coincide com o ponto O quando a soma

Re: [obm-l] Problema 2 da OBM U

2017-11-21 Por tôpico Anderson Torres
OPA! Tem um problema no meu problema! Em 18 de novembro de 2017 16:48, Anderson Torres escreveu: > Em 15 de novembro de 2017 15:01, Otávio Araújo > escreveu: >> Alguém poderia me ajudar no problema 2 da segunda fase da obm u desse ano? O

Re: [obm-l] Problema 2 da OBM U

2017-11-18 Por tôpico Anderson Torres
Em 15 de novembro de 2017 15:01, Otávio Araújo escreveu: > Alguém poderia me ajudar no problema 2 da segunda fase da obm u desse ano? O > enunciado é o seguinte: > "Fixados os inteiros positivos a e b, mostre que o conjunto dos divisores > primos dos termos da sequencia

[obm-l] Re: [obm-l] Problema de álgebra

2017-09-15 Por tôpico Carlos Nehab
Oi, Leonardo (e Ralph) Resolvi postar meu "rabisco de tentativa de solução" pois acho (e com certeza Ralph tb) que isso enriquece o aprendizado da gurizada (sorry pelo gurizada, mas me formei em 1969...). Fiz o seguinte: (Supondo numa primeira abordagem que x, y e z fossem >= -1, prá ver onde

[obm-l] Re: [obm-l] Re: [obm-l] Problema de álgebra

2017-09-15 Por tôpico Leonardo Joau
On Fri, 15 Sep 2017 at 18:42 Ralph Teixeira wrote: > Bom, suponho que queremos alguma solucao que nao use tecnicas de Calculo? > > Que tal assim: x, y e z sao raizes do polinomio: > > t^3-t^2+at-P=0 > > onde P eh o que voce quer maximizar. > > O polinomio f(t)=t^3-t^2+at-P

[obm-l] Re: [obm-l] Problema de álgebra

2017-09-15 Por tôpico Ralph Teixeira
Bom, suponho que queremos alguma solucao que nao use tecnicas de Calculo? Que tal assim: x, y e z sao raizes do polinomio: t^3-t^2+at-P=0 onde P eh o que voce quer maximizar. O polinomio f(t)=t^3-t^2+at-P sempre tem pelo menos uma raiz real (grau 3). Quando voce muda P, voce translada o

[obm-l] Re: [obm-l] Re: [obm-l] Re: [obm-l] Problema difícil.

2017-09-13 Por tôpico Ralph Teixeira
Oi, Douglas. Acho que o que você fez é um bom começo. Vamos adaptar: pense ao invés nos números de 1009 a 2017 (conjunto A). i) Eles podem todos parear com os números de 1 a 1008? ii) Então pelo menos um produto usando os elementos de A vai dar NO MÍNIMO NO MÍNIMO... iii) Esse número do item

[obm-l] Re: [obm-l] Re: [obm-l] Problema difícil.

2017-09-13 Por tôpico Douglas Oliveira de Lima
Então Bernardo, eu pensei numa parada mas não tenho certeza , pensei que os números 997,998,999,...,1994 Não poderiam ocupar as posições de 1 a 1997, logo pelo menos um deles ocuparia uma posição não inferior a 998, aí pensei no 997.998=995006. Em 12 de set de 2017 18:39, "Bernardo Freitas Paulo

[obm-l] Re: [obm-l] Re: [obm-l] Re: [obm-l] Problema difícil.

2017-09-12 Por tôpico Israel Meireles Chrisostomo
Exatamente, aplique a desigualdade do rearranjo Em 12 de setembro de 2017 19:08, Leonardo Joau escreveu: > > On Tue, 12 Sep 2017 at 18:39 Bernardo Freitas Paulo da Costa < > bernardo...@gmail.com> wrote: > >> 2017-09-12 17:51 GMT-03:00 Douglas Oliveira de Lima >>

[obm-l] Re: [obm-l] Re: [obm-l] Problema difícil.

2017-09-12 Por tôpico Leonardo Joau
On Tue, 12 Sep 2017 at 18:39 Bernardo Freitas Paulo da Costa < bernardo...@gmail.com> wrote: > 2017-09-12 17:51 GMT-03:00 Douglas Oliveira de Lima > : > > Considere a sequência de números 1,2,3,4,5,...,2017. > > E uma certa ordenação deles a1, a2, a3, ..., a2017. >

[obm-l] Re: [obm-l] Problema difícil.

2017-09-12 Por tôpico Bernardo Freitas Paulo da Costa
2017-09-12 17:51 GMT-03:00 Douglas Oliveira de Lima : > Considere a sequência de números 1,2,3,4,5,...,2017. > E uma certa ordenação deles a1, a2, a3, ..., a2017. > Agora multiplique respectivamente os números das duas sequencias > determinando assim uma nova

Re: [obm-l] Problema estranho

2017-09-07 Por tôpico Anderson Torres
Bora lá... Pelo que a galera já demonstrou, o resultado vale se todos os números da sequência forem racionais. Agora, falta cobrir os irracionais. Considere - real eps>0 - inteiro m>0 - inteiros p_1, p_2, ... p_(2n+1) tais que, para todo i, vale |p_i-mx_i| < eps. A ideia é que se eps for bem

Re: [obm-l] Problema de grafos

2017-09-03 Por tôpico Daniel da Silva
Obrigado pela ajuda Esdras e Matheus. Daniel Rocha da Silva > Em 2 de set de 2017, às 13:23, Esdras Muniz > escreveu: > > Cada vértice pode ter como grau um número de 0 a n-1, porém o 0 e o n-1 > não podem ambos ser graus de vértices, pois se um tem grau n-1

Re: [obm-l] Problema de grafos

2017-09-02 Por tôpico Esdras Muniz
Cada vértice pode ter como grau um número de 0 a n-1, porém o 0 e o n-1 não podem ambos ser graus de vértices, pois se um tem grau n-1 então ele está ligado a todos os outros vértices. Então há apenas n-1 possibilidades para o grau de cada vértice. Pelo pcp há dois vértices com o mesmo grau. Em 2

Re: [obm-l] Problema de grafos

2017-09-02 Por tôpico Matheus Secco
Olá Daniel, veja que os graus podem variar de 0 até n - 1. Entretanto, não é possível ter um vértice com grau 0 e outro com grau n - 1. Desta forma, em vez de n possibilidades para o grau de cada vértice, há n - 1 possibilidades para o grau de cada vértice. Como há n vértices, pelo Princípio da

Re: [obm-l] Problema de Probabilidade

2017-08-08 Por tôpico Ralph Teixeira
Ah, se voce preferir, pode dividir a tabela por jogador mesmo, assim: /// A B CD E FG Total JV 60 60 60 60 45 45 25 355 JP 40 40 40 40 55 55 75 345 Tot 100 100 100 100 100 100 100 700 a) Pr(JV)=355/700 b) Pr(E|JV)=45/355 Abraco, Ralph.

[obm-l] Re: [obm-l] Problema de função elementar

2017-07-17 Por tôpico Alexandre Antunes
Boa noite, Encontrei um resultado aproximado F (21/2017)=F(0,01)=0,054719 Não sei se fiz "besteiras", mas usando a expressão em b F (x/3) = F(x)/2 x=1 =》F(1/3)=F(1)/2=1/2 x=1/3 =》F(1/9)=F(1/3)/2=1/4 Generalizando x=1/3^n =》F(1/3^n)=1/2^n Por outro lado Para

[obm-l] Re: [obm-l] Re: [obm-l] Re: [obm-l] Problema de função elementar

2017-07-17 Por tôpico Guilherme Oliveira
Aproveitando o problema, quanto seria f (0,1)? Tenham uma boa noite, Guilherme Em 17/07/2017 12:45, "Pedro José" escreveu: Bom dia! Seguindo a linha proposta pelo Anderson. 7/3^6 < 21/2017 < 8/3^6 ==> F(21/2017)= F(7/3^6)=F(8/3^6) F(7/9) = 3/4. F(7/3^6) = F(7/9/3^4)=

[obm-l] Re: [obm-l] Re: [obm-l] Problema de função elementar

2017-07-17 Por tôpico Pedro José
Bom dia! Seguindo a linha proposta pelo Anderson. 7/3^6 < 21/2017 < 8/3^6 ==> F(21/2017)= F(7/3^6)=F(8/3^6) F(7/9) = 3/4. F(7/3^6) = F(7/9/3^4)= F(7/9)/2^4= 3/2^6= 3/64. Sds, PJMS Em 17 de julho de 2017 10:48, Pedro José escreveu: > Bom dia! > > Há uma restrição para

[obm-l] Re: [obm-l] Re: [obm-l] Problema de função elementar

2017-07-17 Por tôpico Pedro José
Bom dia! Há uma restrição para a função ser crescente. Portanto F(1) é máximo e F(1) = 1, logo não pode ser 87. tem que ser um valor menor ou igual a 1 e maior ou igual a zero. Sds, PJMS Em 15 de julho de 2017 20:54, Matheus Herculano < matheusherculan...@gmail.com> escreveu: > O resultado é

[obm-l] Re: [obm-l] Problema de função elementar

2017-07-15 Por tôpico Matheus Herculano
O resultado é 87 Em 13 de jul de 2017 09:51, "Douglas Oliveira de Lima" < profdouglaso.del...@gmail.com> escreveu: > Seja F uma função crescente definida para todo número real x, 0<=x<=1, tal > que > > a) F(0)=0 > > b) F(x/3)=F(x)/2 > > c) F(1-x)=1-F(x) > > Encontrar F(21/2017). >

[obm-l] Re: [obm-l] Problema de função elementar

2017-07-15 Por tôpico Anderson Torres
F(1) = 1 F(1/3)=1/2, F(2/3)=1/2 - logo, F(x) = 1/2 se x está em [1/3,2/3] F(1/9)=1/4, F(2/9)=1/4, F(7/9)=3/4, F(8/9)=3/4 logo, F(x) = 1/4 se x está em [1/9,2/9] e F(x) = 3/4 se x está em [7/9,8/9] Acho que a ideia é por aí: ver em que terço-médio cairá o valor 21/2017. Em 13 de julho de 2017

Re: [obm-l] Problema estranho

2017-07-15 Por tôpico Anderson Torres
Uma ideia pode ser tentar aproximar os reais para racionais e usar o argumento das potências, não? Em 11 de julho de 2017 18:21, Matheus Secco escreveu: > Oi Ralph, tava sem tempo de escrever, mas vou aproveitar a deixa porque você > já fez quase tudo. Acho que dá pra

Re: [obm-l] Problema estranho

2017-07-11 Por tôpico Matheus Secco
Oi Ralph, tava sem tempo de escrever, mas vou aproveitar a deixa porque você já fez quase tudo. Acho que dá pra fazer o caso geral usando que os reais admitem uma base considerando como um espaço vetorial sobre os racionais. Em ter, 11 de jul de 2017 às 18:18, Ralph Teixeira

Re: [obm-l] Problema estranho

2017-07-11 Por tôpico Ralph Teixeira
Ah, melhor ainda: depois que seus números forem inteiros, some uma certa constante a todos eles de forma que um deles seja 0. Agora divida por 2, quantas vezes você quiser (eles vão ser sempre todos pares pelo argumento de paridade anterior!). Então são todos inteiros divisíveis por poências

Re: [obm-l] Problema estranho

2017-07-11 Por tôpico Ralph Teixeira
Bom, eu sei resolver se todos os números forem racionais. Deve ter um jeito de usar isso para o caso geral... A propriedade desse conjunto não se altera se todos os elementos do conjunto forem multiplicados por um mesmo número, nem se a gente somar uma certa constante a todos eles. Assim, *SE*

Re: [obm-l] Problema estranho

2017-07-11 Por tôpico Nowras Ali
Uma prova por indução me parece o melhor caminho. O Bernardo já provou para o caso base, basta agora tentar provar para n+1, assumindo verdadeiro para n. Tentarei resolver o problema assim que puder. Abraços, Nowras. Em 9 de julho de 2017 18:54, Otávio Araújo

Re: [obm-l] Problema estranho

2017-07-09 Por tôpico Otávio Araújo
Já tentei isso, porém não parece ajudar em muita coisa mas de qualquer forma obrigado > Em 9 de jul de 2017, às 18:00, Bernardo Freitas Paulo da Costa > escreveu: > > Não pensei muito, mas acho que você deveria tentar provar os casos n=1 > e n=2 "no braço" para

Re: [obm-l] Problema estranho

2017-07-09 Por tôpico Bernardo Freitas Paulo da Costa
Não pensei muito, mas acho que você deveria tentar provar os casos n=1 e n=2 "no braço" para ter a intuição. E, na verdade, o enunciado deveria ser: dados a_1, a_2, ... a_{2n+1} números reais, não necessariamente distintos, tais que, para cada escolha de 2n dentre eles, é possível separar em dois

Re: [obm-l] Problema estranho

2017-07-09 Por tôpico Luiz Antonio Rodrigues
Olá, Francisco! Eu também pensei nisso, mas vou consultar o site que o Bruno indicou... Muito obrigado e um abraço! Luiz On Jul 8, 2017 9:13 PM, "Francisco Barreto" wrote: > > On Sat, 8 Jul 2017 at 20:21 Otávio Araújo > wrote: > >> >> O

Re: [obm-l] Problema estranho

2017-07-09 Por tôpico Luiz Antonio Rodrigues
Olá, Bruno! Muito obrigado pelo esclarecimento! Um abraço! Luiz On Jul 8, 2017 8:01 PM, "Bruno Visnadi" wrote: > Tecnicamente não dá para chamar de conjunto, quando há números repetidos. > O correto seria Multiconjunto: https://pt.wikipedia.org/wiki/Multiconjunto >

Re: [obm-l] Problema estranho

2017-07-08 Por tôpico Francisco Barreto
On Sat, 8 Jul 2017 at 20:21 Otávio Araújo wrote: > > O enunciado original eu não vi, quem me falou desse problema foi um amigo > meu. assim me perdoe pelo erro grosseiro. Mas considerando esse A um > multiconjunto, essa questão é verdadeira ou se tem um contra-exemplo?

Re: [obm-l] Problema estranho

2017-07-08 Por tôpico Francisco Barreto
On Sat, 8 Jul 2017 at 17:35 Otávio Araújo wrote: > Galera, queria que alguém pudesse resolver essa questão pra mim ( passei > muito tempo nela já kkk): > " Seja n um natural positivo e A um conjunto de 2n+1 números reais, não > necessariamente distintos, com a

Re: [obm-l] Problema estranho

2017-07-08 Por tôpico Otávio Araújo
O enunciado original eu não vi, quem me falou desse problema foi um amigo meu. assim me perdoe pelo erro grosseiro. Mas considerando esse A um multiconjunto, essa questão é verdadeira ou se tem um contra-exemplo? > Em 8 de jul de 2017, às 19:47, Bruno Visnadi >

Re: [obm-l] Problema estranho

2017-07-08 Por tôpico Bruno Visnadi
Tecnicamente não dá para chamar de conjunto, quando há números repetidos. O correto seria Multiconjunto: https://pt.wikipedia.org/wiki/Multiconjunto Em 8 de julho de 2017 19:27, Luiz Antonio Rodrigues escreveu: > Olá, Otávio! > Desculpe a intromissão. Eu não sei como

Re: [obm-l] Problema estranho

2017-07-08 Por tôpico Luiz Antonio Rodrigues
Olá, Otávio! Desculpe a intromissão. Eu não sei como resolver seu problema, mas quero aproveitá-lo para colocar uma questão que me atormenta desde a faculdade: pode existir um conjunto {1,1,1,2,3}? O número 1 não é único? Um abraço! Luiz On Jul 8, 2017 5:35 PM, "Otávio Araújo"

  1   2   3   4   5   6   7   8   9   10   >